Direct sum of isomorphic simple modulesA ring without the Invariant Basis Number propertyIn a unital...

Copycat chess is back

Why is the design of haulage companies so “special”?

What do you call a Matrix-like slowdown and camera movement effect?

Draw simple lines in Inkscape

Should I join office cleaning event for free?

Why is an old chain unsafe?

The use of multiple foreign keys on same column in SQL Server

How to calculate implied correlation via observed market price (Margrabe option)

Are tax years 2016 & 2017 back taxes deductible for tax year 2018?

Do airline pilots ever risk not hearing communication directed to them specifically, from traffic controllers?

What makes Graph invariants so useful/important?

Is there a minimum number of transactions in a block?

Example of a relative pronoun

least quadratic residue under GRH: an EXPLICIT bound

What are these boxed doors outside store fronts in New York?

I probably found a bug with the sudo apt install function

Japan - Plan around max visa duration

N.B. ligature in Latex

Can I interfere when another PC is about to be attacked?

How can I fix this gap between bookcases I made?

How old can references or sources in a thesis be?

Is it possible to do 50 km distance without any previous training?

Download, install and reboot computer at night if needed

Why are 150k or 200k jobs considered good when there are 300k+ births a month?



Direct sum of isomorphic simple modules


A ring without the Invariant Basis Number propertyIn a unital $R$-module $M$, if $forall M_1!lneq!M;;exists M_2!lneq!M$, such that $M_1!cap!M_2!=!{0}$, then $M$ is semisimplemodules is direct sum of simple submodule all of which are isomorphic to simple moduleProve equivalence: Internal Direct Sum of Modules (HW)Is there a direct proof that $M_n(mathcal k)$ is semisimple ring.Confused about simple and semisimple modulesCharacterize semisimple rings of quotientswhy is a simple ring not semisimple?A direct sum of semisimple left R-modules is semisimpleAn infinite direct product of simple module over semisimple ringFinite inner direct sums of simple modules













2












$begingroup$


Let $R$ be a ring and $M$ an $R$-module.
Suppose that ${M_i}_{iin I}$ is a (possibly infinite) collection of simple submodules of $M$, which are pairwise isomorphic.
Suppose that $M$ is the direct sum of ${M_i}_{iin I}$.
Suppose further that $M$ is also the direct sum of ${K_j}_{jin J}$, where each $K_j$ is a simple submodule of $M$.



I can prove that each $K_j$ is isomorphic to each $M_i$. However, is it true that $I$ and $J$ are of the same cardinality? If so, I'd appreciate a direct proof (using basic equivalent definitions of a semisimple module is fine).










share|cite|improve this question









$endgroup$












  • $begingroup$
    It's clearly true if $I$ is finite, so we can restrict to $|I|geqaleph_0$. I have some things I think are true when $I$ is infinite, but it will take a better set-theorist than myself to justify it. The intuition I have is that if $|I|leq |M_i|$, then $|oplus_{iin I}M_i|=|M_i|$ and that if $|I|> |M_i|$, $|oplus_{iin I}M_i|=|I|$. Please take this with a grain of salt, because I am not 100% sure both these facts are true.
    $endgroup$
    – rschwieb
    Mar 20 at 13:46












  • $begingroup$
    I would assume it is, and I would proof it by testing it with a copy of $M_j cong K_i$, since you know precisely the form of those morphism spaces ($k J$ and $k I$ where $k X$ is the free vectorspace on $X$).
    $endgroup$
    – Enkidu
    Mar 20 at 13:53






  • 1




    $begingroup$
    If your ring is commutative, it follows from uniqueness of cardinality of bases over vector spaces, right ? @rschwieb : your cardinality computations are correct, in fact in general $|oplus_i M_i| = displaystylesum_n sum_{Esubset I, |E| = n} |M_i|^n$, so when $|M_i|$ is finite this is just the cardinality of the set of finite subsets of $I$, which is just $|I|$, and $|M_i|$ is infinite, this is just $|I||M_i| = max {|I|, |M_i|}$
    $endgroup$
    – Max
    Mar 20 at 13:57












  • $begingroup$
    We can get even more precise by noting that $hom(M,M_i) = hom (K_j, M_i)^J = hom(M_i, M_i)^I$, so if $kappa = |hom (M_i, M_i)|$ then $kappa^I = kappa^J$. Of course exponentiation of cardinals is a terrible thing, but if $|I|leq |M_i|$ and $kappa$ is small enough this might bring some information
    $endgroup$
    – Max
    Mar 20 at 14:07










  • $begingroup$
    Also note $|hom(M_i, M)|= sum_{Esubset I, mathrm{finite}} |hom (M_i, M_i^E)|= sum_{Esubset I, mathrm{finite}} kappa^E = |I|$ if $kappa$ is finite, $=kappa |I|$ otherwise. So this is more precise : if $kappa leq |I|$ then we get $|I|=|J|$ too (morally, this can happen if $M_i$ has very few endomorphisms, so for instance if $Z(R)$ is very small)
    $endgroup$
    – Max
    Mar 20 at 14:13
















2












$begingroup$


Let $R$ be a ring and $M$ an $R$-module.
Suppose that ${M_i}_{iin I}$ is a (possibly infinite) collection of simple submodules of $M$, which are pairwise isomorphic.
Suppose that $M$ is the direct sum of ${M_i}_{iin I}$.
Suppose further that $M$ is also the direct sum of ${K_j}_{jin J}$, where each $K_j$ is a simple submodule of $M$.



I can prove that each $K_j$ is isomorphic to each $M_i$. However, is it true that $I$ and $J$ are of the same cardinality? If so, I'd appreciate a direct proof (using basic equivalent definitions of a semisimple module is fine).










share|cite|improve this question









$endgroup$












  • $begingroup$
    It's clearly true if $I$ is finite, so we can restrict to $|I|geqaleph_0$. I have some things I think are true when $I$ is infinite, but it will take a better set-theorist than myself to justify it. The intuition I have is that if $|I|leq |M_i|$, then $|oplus_{iin I}M_i|=|M_i|$ and that if $|I|> |M_i|$, $|oplus_{iin I}M_i|=|I|$. Please take this with a grain of salt, because I am not 100% sure both these facts are true.
    $endgroup$
    – rschwieb
    Mar 20 at 13:46












  • $begingroup$
    I would assume it is, and I would proof it by testing it with a copy of $M_j cong K_i$, since you know precisely the form of those morphism spaces ($k J$ and $k I$ where $k X$ is the free vectorspace on $X$).
    $endgroup$
    – Enkidu
    Mar 20 at 13:53






  • 1




    $begingroup$
    If your ring is commutative, it follows from uniqueness of cardinality of bases over vector spaces, right ? @rschwieb : your cardinality computations are correct, in fact in general $|oplus_i M_i| = displaystylesum_n sum_{Esubset I, |E| = n} |M_i|^n$, so when $|M_i|$ is finite this is just the cardinality of the set of finite subsets of $I$, which is just $|I|$, and $|M_i|$ is infinite, this is just $|I||M_i| = max {|I|, |M_i|}$
    $endgroup$
    – Max
    Mar 20 at 13:57












  • $begingroup$
    We can get even more precise by noting that $hom(M,M_i) = hom (K_j, M_i)^J = hom(M_i, M_i)^I$, so if $kappa = |hom (M_i, M_i)|$ then $kappa^I = kappa^J$. Of course exponentiation of cardinals is a terrible thing, but if $|I|leq |M_i|$ and $kappa$ is small enough this might bring some information
    $endgroup$
    – Max
    Mar 20 at 14:07










  • $begingroup$
    Also note $|hom(M_i, M)|= sum_{Esubset I, mathrm{finite}} |hom (M_i, M_i^E)|= sum_{Esubset I, mathrm{finite}} kappa^E = |I|$ if $kappa$ is finite, $=kappa |I|$ otherwise. So this is more precise : if $kappa leq |I|$ then we get $|I|=|J|$ too (morally, this can happen if $M_i$ has very few endomorphisms, so for instance if $Z(R)$ is very small)
    $endgroup$
    – Max
    Mar 20 at 14:13














2












2








2


1



$begingroup$


Let $R$ be a ring and $M$ an $R$-module.
Suppose that ${M_i}_{iin I}$ is a (possibly infinite) collection of simple submodules of $M$, which are pairwise isomorphic.
Suppose that $M$ is the direct sum of ${M_i}_{iin I}$.
Suppose further that $M$ is also the direct sum of ${K_j}_{jin J}$, where each $K_j$ is a simple submodule of $M$.



I can prove that each $K_j$ is isomorphic to each $M_i$. However, is it true that $I$ and $J$ are of the same cardinality? If so, I'd appreciate a direct proof (using basic equivalent definitions of a semisimple module is fine).










share|cite|improve this question









$endgroup$




Let $R$ be a ring and $M$ an $R$-module.
Suppose that ${M_i}_{iin I}$ is a (possibly infinite) collection of simple submodules of $M$, which are pairwise isomorphic.
Suppose that $M$ is the direct sum of ${M_i}_{iin I}$.
Suppose further that $M$ is also the direct sum of ${K_j}_{jin J}$, where each $K_j$ is a simple submodule of $M$.



I can prove that each $K_j$ is isomorphic to each $M_i$. However, is it true that $I$ and $J$ are of the same cardinality? If so, I'd appreciate a direct proof (using basic equivalent definitions of a semisimple module is fine).







abstract-algebra ring-theory modules representation-theory






share|cite|improve this question













share|cite|improve this question











share|cite|improve this question




share|cite|improve this question










asked Mar 20 at 12:27









Chris AChris A

133




133












  • $begingroup$
    It's clearly true if $I$ is finite, so we can restrict to $|I|geqaleph_0$. I have some things I think are true when $I$ is infinite, but it will take a better set-theorist than myself to justify it. The intuition I have is that if $|I|leq |M_i|$, then $|oplus_{iin I}M_i|=|M_i|$ and that if $|I|> |M_i|$, $|oplus_{iin I}M_i|=|I|$. Please take this with a grain of salt, because I am not 100% sure both these facts are true.
    $endgroup$
    – rschwieb
    Mar 20 at 13:46












  • $begingroup$
    I would assume it is, and I would proof it by testing it with a copy of $M_j cong K_i$, since you know precisely the form of those morphism spaces ($k J$ and $k I$ where $k X$ is the free vectorspace on $X$).
    $endgroup$
    – Enkidu
    Mar 20 at 13:53






  • 1




    $begingroup$
    If your ring is commutative, it follows from uniqueness of cardinality of bases over vector spaces, right ? @rschwieb : your cardinality computations are correct, in fact in general $|oplus_i M_i| = displaystylesum_n sum_{Esubset I, |E| = n} |M_i|^n$, so when $|M_i|$ is finite this is just the cardinality of the set of finite subsets of $I$, which is just $|I|$, and $|M_i|$ is infinite, this is just $|I||M_i| = max {|I|, |M_i|}$
    $endgroup$
    – Max
    Mar 20 at 13:57












  • $begingroup$
    We can get even more precise by noting that $hom(M,M_i) = hom (K_j, M_i)^J = hom(M_i, M_i)^I$, so if $kappa = |hom (M_i, M_i)|$ then $kappa^I = kappa^J$. Of course exponentiation of cardinals is a terrible thing, but if $|I|leq |M_i|$ and $kappa$ is small enough this might bring some information
    $endgroup$
    – Max
    Mar 20 at 14:07










  • $begingroup$
    Also note $|hom(M_i, M)|= sum_{Esubset I, mathrm{finite}} |hom (M_i, M_i^E)|= sum_{Esubset I, mathrm{finite}} kappa^E = |I|$ if $kappa$ is finite, $=kappa |I|$ otherwise. So this is more precise : if $kappa leq |I|$ then we get $|I|=|J|$ too (morally, this can happen if $M_i$ has very few endomorphisms, so for instance if $Z(R)$ is very small)
    $endgroup$
    – Max
    Mar 20 at 14:13


















  • $begingroup$
    It's clearly true if $I$ is finite, so we can restrict to $|I|geqaleph_0$. I have some things I think are true when $I$ is infinite, but it will take a better set-theorist than myself to justify it. The intuition I have is that if $|I|leq |M_i|$, then $|oplus_{iin I}M_i|=|M_i|$ and that if $|I|> |M_i|$, $|oplus_{iin I}M_i|=|I|$. Please take this with a grain of salt, because I am not 100% sure both these facts are true.
    $endgroup$
    – rschwieb
    Mar 20 at 13:46












  • $begingroup$
    I would assume it is, and I would proof it by testing it with a copy of $M_j cong K_i$, since you know precisely the form of those morphism spaces ($k J$ and $k I$ where $k X$ is the free vectorspace on $X$).
    $endgroup$
    – Enkidu
    Mar 20 at 13:53






  • 1




    $begingroup$
    If your ring is commutative, it follows from uniqueness of cardinality of bases over vector spaces, right ? @rschwieb : your cardinality computations are correct, in fact in general $|oplus_i M_i| = displaystylesum_n sum_{Esubset I, |E| = n} |M_i|^n$, so when $|M_i|$ is finite this is just the cardinality of the set of finite subsets of $I$, which is just $|I|$, and $|M_i|$ is infinite, this is just $|I||M_i| = max {|I|, |M_i|}$
    $endgroup$
    – Max
    Mar 20 at 13:57












  • $begingroup$
    We can get even more precise by noting that $hom(M,M_i) = hom (K_j, M_i)^J = hom(M_i, M_i)^I$, so if $kappa = |hom (M_i, M_i)|$ then $kappa^I = kappa^J$. Of course exponentiation of cardinals is a terrible thing, but if $|I|leq |M_i|$ and $kappa$ is small enough this might bring some information
    $endgroup$
    – Max
    Mar 20 at 14:07










  • $begingroup$
    Also note $|hom(M_i, M)|= sum_{Esubset I, mathrm{finite}} |hom (M_i, M_i^E)|= sum_{Esubset I, mathrm{finite}} kappa^E = |I|$ if $kappa$ is finite, $=kappa |I|$ otherwise. So this is more precise : if $kappa leq |I|$ then we get $|I|=|J|$ too (morally, this can happen if $M_i$ has very few endomorphisms, so for instance if $Z(R)$ is very small)
    $endgroup$
    – Max
    Mar 20 at 14:13
















$begingroup$
It's clearly true if $I$ is finite, so we can restrict to $|I|geqaleph_0$. I have some things I think are true when $I$ is infinite, but it will take a better set-theorist than myself to justify it. The intuition I have is that if $|I|leq |M_i|$, then $|oplus_{iin I}M_i|=|M_i|$ and that if $|I|> |M_i|$, $|oplus_{iin I}M_i|=|I|$. Please take this with a grain of salt, because I am not 100% sure both these facts are true.
$endgroup$
– rschwieb
Mar 20 at 13:46






$begingroup$
It's clearly true if $I$ is finite, so we can restrict to $|I|geqaleph_0$. I have some things I think are true when $I$ is infinite, but it will take a better set-theorist than myself to justify it. The intuition I have is that if $|I|leq |M_i|$, then $|oplus_{iin I}M_i|=|M_i|$ and that if $|I|> |M_i|$, $|oplus_{iin I}M_i|=|I|$. Please take this with a grain of salt, because I am not 100% sure both these facts are true.
$endgroup$
– rschwieb
Mar 20 at 13:46














$begingroup$
I would assume it is, and I would proof it by testing it with a copy of $M_j cong K_i$, since you know precisely the form of those morphism spaces ($k J$ and $k I$ where $k X$ is the free vectorspace on $X$).
$endgroup$
– Enkidu
Mar 20 at 13:53




$begingroup$
I would assume it is, and I would proof it by testing it with a copy of $M_j cong K_i$, since you know precisely the form of those morphism spaces ($k J$ and $k I$ where $k X$ is the free vectorspace on $X$).
$endgroup$
– Enkidu
Mar 20 at 13:53




1




1




$begingroup$
If your ring is commutative, it follows from uniqueness of cardinality of bases over vector spaces, right ? @rschwieb : your cardinality computations are correct, in fact in general $|oplus_i M_i| = displaystylesum_n sum_{Esubset I, |E| = n} |M_i|^n$, so when $|M_i|$ is finite this is just the cardinality of the set of finite subsets of $I$, which is just $|I|$, and $|M_i|$ is infinite, this is just $|I||M_i| = max {|I|, |M_i|}$
$endgroup$
– Max
Mar 20 at 13:57






$begingroup$
If your ring is commutative, it follows from uniqueness of cardinality of bases over vector spaces, right ? @rschwieb : your cardinality computations are correct, in fact in general $|oplus_i M_i| = displaystylesum_n sum_{Esubset I, |E| = n} |M_i|^n$, so when $|M_i|$ is finite this is just the cardinality of the set of finite subsets of $I$, which is just $|I|$, and $|M_i|$ is infinite, this is just $|I||M_i| = max {|I|, |M_i|}$
$endgroup$
– Max
Mar 20 at 13:57














$begingroup$
We can get even more precise by noting that $hom(M,M_i) = hom (K_j, M_i)^J = hom(M_i, M_i)^I$, so if $kappa = |hom (M_i, M_i)|$ then $kappa^I = kappa^J$. Of course exponentiation of cardinals is a terrible thing, but if $|I|leq |M_i|$ and $kappa$ is small enough this might bring some information
$endgroup$
– Max
Mar 20 at 14:07




$begingroup$
We can get even more precise by noting that $hom(M,M_i) = hom (K_j, M_i)^J = hom(M_i, M_i)^I$, so if $kappa = |hom (M_i, M_i)|$ then $kappa^I = kappa^J$. Of course exponentiation of cardinals is a terrible thing, but if $|I|leq |M_i|$ and $kappa$ is small enough this might bring some information
$endgroup$
– Max
Mar 20 at 14:07












$begingroup$
Also note $|hom(M_i, M)|= sum_{Esubset I, mathrm{finite}} |hom (M_i, M_i^E)|= sum_{Esubset I, mathrm{finite}} kappa^E = |I|$ if $kappa$ is finite, $=kappa |I|$ otherwise. So this is more precise : if $kappa leq |I|$ then we get $|I|=|J|$ too (morally, this can happen if $M_i$ has very few endomorphisms, so for instance if $Z(R)$ is very small)
$endgroup$
– Max
Mar 20 at 14:13




$begingroup$
Also note $|hom(M_i, M)|= sum_{Esubset I, mathrm{finite}} |hom (M_i, M_i^E)|= sum_{Esubset I, mathrm{finite}} kappa^E = |I|$ if $kappa$ is finite, $=kappa |I|$ otherwise. So this is more precise : if $kappa leq |I|$ then we get $|I|=|J|$ too (morally, this can happen if $M_i$ has very few endomorphisms, so for instance if $Z(R)$ is very small)
$endgroup$
– Max
Mar 20 at 14:13










1 Answer
1






active

oldest

votes


















1












$begingroup$

This is a special case of the general Krull-Schmidt-Remak-Azumaya theorem.




Theorem 2.12 (Krull-Schmidt-Remak-Azumaya Theorem) Let $M$ be a module that is a direct sum of modules with local endomorphism rings. Then any two direct sum decompositions of $M$ into indecomposable direct summands are isomorphic.




There is an elementary proof of the special case based on a generalization of the concept of dimension of vector spaces, which you can find, for instance, in Jacobson's “Basic Algebra II”.






share|cite|improve this answer









$endgroup$













  • $begingroup$
    Thank you. I have not been able to find the mentioned proof in Jacobson's book. Do you have a more precise reference inside the book?
    $endgroup$
    – Chris A
    Mar 24 at 13:34












  • $begingroup$
    @ChrisA Section 3.6
    $endgroup$
    – egreg
    Mar 24 at 13:36












Your Answer





StackExchange.ifUsing("editor", function () {
return StackExchange.using("mathjaxEditing", function () {
StackExchange.MarkdownEditor.creationCallbacks.add(function (editor, postfix) {
StackExchange.mathjaxEditing.prepareWmdForMathJax(editor, postfix, [["$", "$"], ["\\(","\\)"]]);
});
});
}, "mathjax-editing");

StackExchange.ready(function() {
var channelOptions = {
tags: "".split(" "),
id: "69"
};
initTagRenderer("".split(" "), "".split(" "), channelOptions);

StackExchange.using("externalEditor", function() {
// Have to fire editor after snippets, if snippets enabled
if (StackExchange.settings.snippets.snippetsEnabled) {
StackExchange.using("snippets", function() {
createEditor();
});
}
else {
createEditor();
}
});

function createEditor() {
StackExchange.prepareEditor({
heartbeatType: 'answer',
autoActivateHeartbeat: false,
convertImagesToLinks: true,
noModals: true,
showLowRepImageUploadWarning: true,
reputationToPostImages: 10,
bindNavPrevention: true,
postfix: "",
imageUploader: {
brandingHtml: "Powered by u003ca class="icon-imgur-white" href="https://imgur.com/"u003eu003c/au003e",
contentPolicyHtml: "User contributions licensed under u003ca href="https://creativecommons.org/licenses/by-sa/3.0/"u003ecc by-sa 3.0 with attribution requiredu003c/au003e u003ca href="https://stackoverflow.com/legal/content-policy"u003e(content policy)u003c/au003e",
allowUrls: true
},
noCode: true, onDemand: true,
discardSelector: ".discard-answer"
,immediatelyShowMarkdownHelp:true
});


}
});














draft saved

draft discarded


















StackExchange.ready(
function () {
StackExchange.openid.initPostLogin('.new-post-login', 'https%3a%2f%2fmath.stackexchange.com%2fquestions%2f3155377%2fdirect-sum-of-isomorphic-simple-modules%23new-answer', 'question_page');
}
);

Post as a guest















Required, but never shown

























1 Answer
1






active

oldest

votes








1 Answer
1






active

oldest

votes









active

oldest

votes






active

oldest

votes









1












$begingroup$

This is a special case of the general Krull-Schmidt-Remak-Azumaya theorem.




Theorem 2.12 (Krull-Schmidt-Remak-Azumaya Theorem) Let $M$ be a module that is a direct sum of modules with local endomorphism rings. Then any two direct sum decompositions of $M$ into indecomposable direct summands are isomorphic.




There is an elementary proof of the special case based on a generalization of the concept of dimension of vector spaces, which you can find, for instance, in Jacobson's “Basic Algebra II”.






share|cite|improve this answer









$endgroup$













  • $begingroup$
    Thank you. I have not been able to find the mentioned proof in Jacobson's book. Do you have a more precise reference inside the book?
    $endgroup$
    – Chris A
    Mar 24 at 13:34












  • $begingroup$
    @ChrisA Section 3.6
    $endgroup$
    – egreg
    Mar 24 at 13:36
















1












$begingroup$

This is a special case of the general Krull-Schmidt-Remak-Azumaya theorem.




Theorem 2.12 (Krull-Schmidt-Remak-Azumaya Theorem) Let $M$ be a module that is a direct sum of modules with local endomorphism rings. Then any two direct sum decompositions of $M$ into indecomposable direct summands are isomorphic.




There is an elementary proof of the special case based on a generalization of the concept of dimension of vector spaces, which you can find, for instance, in Jacobson's “Basic Algebra II”.






share|cite|improve this answer









$endgroup$













  • $begingroup$
    Thank you. I have not been able to find the mentioned proof in Jacobson's book. Do you have a more precise reference inside the book?
    $endgroup$
    – Chris A
    Mar 24 at 13:34












  • $begingroup$
    @ChrisA Section 3.6
    $endgroup$
    – egreg
    Mar 24 at 13:36














1












1








1





$begingroup$

This is a special case of the general Krull-Schmidt-Remak-Azumaya theorem.




Theorem 2.12 (Krull-Schmidt-Remak-Azumaya Theorem) Let $M$ be a module that is a direct sum of modules with local endomorphism rings. Then any two direct sum decompositions of $M$ into indecomposable direct summands are isomorphic.




There is an elementary proof of the special case based on a generalization of the concept of dimension of vector spaces, which you can find, for instance, in Jacobson's “Basic Algebra II”.






share|cite|improve this answer









$endgroup$



This is a special case of the general Krull-Schmidt-Remak-Azumaya theorem.




Theorem 2.12 (Krull-Schmidt-Remak-Azumaya Theorem) Let $M$ be a module that is a direct sum of modules with local endomorphism rings. Then any two direct sum decompositions of $M$ into indecomposable direct summands are isomorphic.




There is an elementary proof of the special case based on a generalization of the concept of dimension of vector spaces, which you can find, for instance, in Jacobson's “Basic Algebra II”.







share|cite|improve this answer












share|cite|improve this answer



share|cite|improve this answer










answered Mar 20 at 17:12









egregegreg

185k1486208




185k1486208












  • $begingroup$
    Thank you. I have not been able to find the mentioned proof in Jacobson's book. Do you have a more precise reference inside the book?
    $endgroup$
    – Chris A
    Mar 24 at 13:34












  • $begingroup$
    @ChrisA Section 3.6
    $endgroup$
    – egreg
    Mar 24 at 13:36


















  • $begingroup$
    Thank you. I have not been able to find the mentioned proof in Jacobson's book. Do you have a more precise reference inside the book?
    $endgroup$
    – Chris A
    Mar 24 at 13:34












  • $begingroup$
    @ChrisA Section 3.6
    $endgroup$
    – egreg
    Mar 24 at 13:36
















$begingroup$
Thank you. I have not been able to find the mentioned proof in Jacobson's book. Do you have a more precise reference inside the book?
$endgroup$
– Chris A
Mar 24 at 13:34






$begingroup$
Thank you. I have not been able to find the mentioned proof in Jacobson's book. Do you have a more precise reference inside the book?
$endgroup$
– Chris A
Mar 24 at 13:34














$begingroup$
@ChrisA Section 3.6
$endgroup$
– egreg
Mar 24 at 13:36




$begingroup$
@ChrisA Section 3.6
$endgroup$
– egreg
Mar 24 at 13:36


















draft saved

draft discarded




















































Thanks for contributing an answer to Mathematics Stack Exchange!


  • Please be sure to answer the question. Provide details and share your research!

But avoid



  • Asking for help, clarification, or responding to other answers.

  • Making statements based on opinion; back them up with references or personal experience.


Use MathJax to format equations. MathJax reference.


To learn more, see our tips on writing great answers.




draft saved


draft discarded














StackExchange.ready(
function () {
StackExchange.openid.initPostLogin('.new-post-login', 'https%3a%2f%2fmath.stackexchange.com%2fquestions%2f3155377%2fdirect-sum-of-isomorphic-simple-modules%23new-answer', 'question_page');
}
);

Post as a guest















Required, but never shown





















































Required, but never shown














Required, but never shown












Required, but never shown







Required, but never shown

































Required, but never shown














Required, but never shown












Required, but never shown







Required, but never shown







Popular posts from this blog

Magento 2 - Add success message with knockout Planned maintenance scheduled April 23, 2019 at 23:30 UTC (7:30pm US/Eastern) Announcing the arrival of Valued Associate #679: Cesar Manara Unicorn Meta Zoo #1: Why another podcast?Success / Error message on ajax request$.widget is not a function when loading a homepage after add custom jQuery on custom themeHow can bind jQuery to current document in Magento 2 When template load by ajaxRedirect page using plugin in Magento 2Magento 2 - Update quantity and totals of cart page without page reload?Magento 2: Quote data not loaded on knockout checkoutMagento 2 : I need to change add to cart success message after adding product into cart through pluginMagento 2.2.5 How to add additional products to cart from new checkout step?Magento 2 Add error/success message with knockoutCan't validate Post Code on checkout page

Fil:Tokke komm.svg

Where did Arya get these scars? Unicorn Meta Zoo #1: Why another podcast? Announcing the arrival of Valued Associate #679: Cesar Manara Favourite questions and answers from the 1st quarter of 2019Why did Arya refuse to end it?Has the pronunciation of Arya Stark's name changed?Has Arya forgiven people?Why did Arya Stark lose her vision?Why can Arya still use the faces?Has the Narrow Sea become narrower?Does Arya Stark know how to make poisons outside of the House of Black and White?Why did Nymeria leave Arya?Why did Arya not kill the Lannister soldiers she encountered in the Riverlands?What is the current canonical age of Sansa, Bran and Arya Stark?